NU470 Week 4 PrepU: Ethics

Réussis tes devoirs et examens dès maintenant avec Quizwiz!

A nurse is conducting a monthly chart audit in the newborn nursery. Which finding could indicate an ethical consideration of practice? o 200% increase in IV cath attempts o Overall 2% decrease in length of stay o Change in bottle feeding protocol o 7% increase in the number of clients

o 200% increase in IV cath attempts · With such a significant increase in the number of IV catheter attempts versus the number of clients, this warrants a further investigation as to why such a large change. This could be an ethical consideration in weighing the amount of pain the infant experiences during the procedure.

Which is also known as a proxy directive? o Durable power of attorney for health care o Treatment directive o Living will o Medical directive

o Durable power of attorney for health care · A durable power of attorney for health care is also known as a health care power of attorney or a proxy directive. The other options are incorrect.

An oncology client in an outpatient chemotherapy clinic asks several questions regarding care and treatment. The nurse explains the clinic's routine, typical side effects of the chemotherapy, and ways to decrease the number of side effects experienced. Which characteristic is the nurse demonstrating? o veracity o autonomy o fidelity o justice

o veracity · The nurse in this scenario is demonstrating veracity despite the difficult information she is sharing with the client. Veracity means telling the truth, which is essential to the integrity of the client-provider relationship. Fidelity reflects honoring promises made while justice is related to fairness. Autonomy reflects independence.

The nurse moves a confused, disruptive patient to a private room at the end of the hall so that other patients can rest, even though the confused patient becomes more agitated. The nurse's intervention is consistent with what moral theory? o "Consequentialism," by which good consequences for the greatest number are maximized o "Veracity," in which the nurse has an obligation to tell the truth o "Duty of obligation," by which an action, regardless of its results, is justified if the decision making was based on moral principles o "Paternalism," in which the action limits the patient's autonomy

o "Consequentialism," by which good consequences for the greatest number are maximized · One classic theory in ethics is teleologic theory or consequentialism, which focuses on the ends or consequences of actions. The best-known form of this theory, utilitarianism, is based on the concept of "the greatest good for the greatest number." The choice of action is clear under this theory, because the action that maximizes good over bad is the correct one. The theory poses difficulty when one must judge intrinsic values and determine whose good is the greatest. In addition, it is important to ask whether good consequences can justify any amoral actions that might be used to achieve them.

A hospital employee asks the nurse if another hospital employee is a client on the medical unit. What statements made by the nurse protect client privacy? Select all that apply. o "I am not able to provide that information." o "The client is in room 313." o "Client privacy is part of the hospital code of conduct." o "You should know better than to ask that question." o "You will need to ask your manager."

o "I am not able to provide that information." o "Client privacy is part of the hospital code of conduct." · The two statements: I am not able to provide that information and client privacy is part of the hospital code of conduct are honest statements that do not breach privacy. The two statements: you should know better than to ask that question and you will need to ask your manager are reprimanding the asking employee. Telling the asking employee that the client is in a specific room breaks privacy and confirms what the employee is asking.

A client recently diagnosed with pancreatic cancer asks the nurse not to share the diagnosis with the client's family members. After visiting the client, the client's daughter approaches the nurse and states, "Mom just did not seem herself today. Are biopsy reports back and do they confirm pancreatic cancer?" What is the best response from the nurse to the client's daughter? o "It is unethical and illegal for me to discuss your mother's medical information with you." o "It is unethical and illegal for me to give you the biopsy results; please ask your mother." o "It is unethical of me to discuss biopsy results with anyone but the client involved." o "It is illegal for me to discuss biopsy results with anyone but the client involved."

o "It is unethical and illegal for me to discuss your mother's medical information with you." · Providing a firm response in explaining the need to protect patient information is one strategy to aid the nurse in ethical decision-making. The U.S. Department of Health and Human Services (DHHS) provides for patient confidentiality. Violations of a patient's confidentiality could result in criminal or civil litigation. While it is unethical/illegal to discuss biopsy results with the daughter, statements by the nurse indicating biopsy results are back but cannot be shared indirectly provide the daughter with confidential information. Encouraging the daughter to ask her mother about the biopsy results indirectly provides the daughter with information that the mother knows the biopsy results.

A nurse is accompanying a client to the mall to do some shopping. A neighbor of the nurse approaches and tries to engage the nurse in conversation. What would be the most appropriate response by the nurse to the neighbor? o "I am working at the moment. I will telephone you later." o "Now is not a good time to talk. I will telephone you later." o "My client and I are busy, but I can speak with you another time." o "Hi. Let me introduce you to my client. We are shopping together."

o "Now is not a good time to talk. I will telephone you later." · The nurse must not breach confidentiality. Indicating that the person accompanying the nurse is a client, even by saying "I am working," would be considered a breach of confidentiality.

The nurse is caring for a hospitalized client who is suspicious and guarded. The client tells the nurse that the client does not want anyone to tell the family about the client's condition. What is the nurse's best response when the family calls the hospital unit to inquire about the client's condition? o "I am sorry, the client does not want you included in care." o "It would be best if you came to visit the client here." o "Unfortunately, you are not permitted to call the hospital unit." o "You are welcome to share any information that you think would be helpful."

o "You are welcome to share any information that you think would be helpful." · The client does not want the family to know information regarding their condition. The client's wishes must be upheld unless there is the threat of harm, in which case information can be shared without the client's consent. In this context, however, that is not the case. The nurse's best response is to ask the family to share anything that they think would inform the assessment and treatment of the client.

A nurse is working as part of a research team evaluating children for participation in a research study. The nurse demonstrates respect when the nurse obtains assent from children over which age? o 7 o 16 o 18 o 12

o 7 · Respect for persons requires that the subject is able to independently and autonomously volunteer to participate in a study. Because infants and children do not have this ability, parents must consent on behalf of their children. Cognitively intact children over the age of 7 years should always be asked to give their assent.

The nurse knows that children can give assent (agree) to participate in research. At what intellectual age does the American Academy of Pediatrics suggest minors may first do this? o 7 years o 9 years o 12 years o 14 years

o 7 years · The age of 7 years is considered competent to provide assent when a developmentally appropriate explanation has been given. A child older than 14 years is considered a mature minor in many states.

An employee health nurse is assisting a stressed working mother with value clarification. Which best defines value clarification? o A systematic inquiry into principles of right and wrong conduct, of virtue and vice, and of good and evil, as they relate to conduct. o An organization of values in which each is ranked along a continuum of importance, often leading to a personal code of conduct. o A process by which people come to understand their own values and value systems. o A belief about the worth of something, about what matters, that acts as a standard to guide one's behavior.

o A process by which people come to understand their own values and value systems. · Value clarification is a process by which people come to understand their own values and value systems. A value is a belief about the worth of something, about what matters, that acts as a standard to guide one's behavior. A value system is an organization of values in which each is ranked along a continuum of importance, often leading to a personal code of conduct. Ethics is a systematic inquiry into principles of right and wrong conduct, of virtue and vice, and of good and evil, as they relate to conduct.

The nurse is caring for a young adult with end stage leukemia. The client asks the nurse to "help end my suffering in this life because it has gotten to be too much to endure." Based on the ANA Code of Ethics for nurses, what would the nurse do next? Select all that apply. o Ask the family members to talk to the client about his desires to end his life. o Allow the client to discuss their feelings and explore other options for comfort. o Explain to the client that nurses cannot participate in assisted suicide. o Administer the client's next scheduled morphine sulfate dose early. o Tell the client they are just depressed and ask the health care provider for an antidepressant.

o Allow the client to discuss their feelings and explore other options for comfort. o Explain to the client that nurses cannot participate in assisted suicide. · The American Nurses Association Position Statement on Assisted Suicide acknowledges the complexity of the assisted suicide debate, but clearly states that nursing participation in assisted suicide is a violation of the Code for Nurses. Legally, nurses are not allowed to administer medications even if prescribed by a physician if that medication will hasten the client's end of life. A nurse should encourage clients to discuss their feelings and assist them to explore other options. The client may need an antidepressant, but this does not deal with the current issue of suicidal ideations.

During a nursing shift, which events warrant completion of an incident report? Select all that apply. o An intravenous antibiotic was administered 2 hours late because the IV site infiltrated. o A visitor slipped and fell in the hallway, but was not injured. o A nurse asks an unlicensed assistive personnel (UAP) to feed a client. o A client falls while being transferred from the bed to the chair. o A nurse reports that a client is crying and distraught over a diagnosis of metastatic cancer.

o An intravenous antibiotic was administered 2 hours late because the IV site infiltrated. o A visitor slipped and fell in the hallway, but was not injured. o A client falls while being transferred from the bed to the chair. · An incident report is a written account of an unusual, potentially injurious event involving a client, employee, or visitor. Late administration of medication is considered a medication error and is potentially injurious to the client. A visitor fall and a client fall are both reportable situations. A client crying following a diagnosis of cancer could be expected, and a nurse delegating appropriate care to a UAP is not reportable. Therefore, these actions do not require an incident report to be filed.

Which is not considered a step in the values clarification process? o Assessing o Prizing o Acting o Choosing

o Assessing · Assessing is not a step in the values clarification process. Choosing is when the person considers a range of possibilities and freely chooses the values that feel right. Prizing is when the person considered the value, cherishes it, and publicly attaches it to himself or herself. Acting is when the person puts the values into action.

A new nursing student is studying ethics in nursing and informs a client who wants to stop medication about its benefits and how the client will continue to feel better only if use of the drug continues. Which concept is the nursing student using? o Justice o Beneficence o Autonomy o Veracity

o Beneficence · Beneficence is the principle of using the knowledge of science and incorporating the art of caring to develop an environment in which individuals achieve their maximal health care potential. It is "doing good."

Using the nursing process to make ethical decisions involves following several steps. Which step is the nurse implementing when reflecting on the decision-making process and the role it will play in making future decisions? o Implementing o Diagnosing o Evaluating o Planning

o Evaluating · Evaluating an ethical decision involves reflecting on the process and evaluating those elements that will be helpful in the future. The nurse may also question how this experience can improve reasoning and decision making in the future. Diagnosing the ethical problem involves stating the problem clearly. Planning includes identifying the options and exploring the probable short-term and long-term consequences. Implementing includes the implementation of the decision and comparing the outcomes of the action with what was considered and hoped for in advance.

A nurse is searching the Internet for information about theorists associated with moral development. The search reveals several theorists. Which result would the nurse identify as being associated with women and morality? o Fowler o Piaget o Gilligan o Kohlberg

o Gilligan · Gilligan (1977; 1982) originally worked with Kohlberg. As she listened to women discuss their own real-life moral conflicts, she recognized that there was a conception of morality from the female viewpoint that was not represented in Kohlberg's work. Gilligan's theory views girls and women as developing a morality of response and care, and boys and men as developing a morality of justice. Piaget developed a theory of cognitive development. Fowler postulated a developmental theory of the spiritual identity of humans.

The mental health nurse is responsible for maintaining professional boundaries. Which would be an example of a professional boundary violation? o Giving personalized gifts to a client o Exhibiting confidentiality o Avoiding personal attachment to the client o Providing a friendly environment

o Giving personalized gifts to a client · An example of a professional boundary violation includes giving personal gifts to a client. Providing a friendly environment, exhibiting confidentiality, and avoiding personal attachment to the client are not boundary violations.

A parent of a 17-year-old high school student is allowing the child to decide which college the child will attend. When the child requests direction from the parent in making this decision, the parent responds by stating, "You will need to make this decision on your own." What type of value transmission is the parent displaying? o Laissez-faire o Responsible choice o Modeling o Moralizing

o Laissez-faire · This situation demonstrates laissez-faire value transmission, which is characterized by allowing the adolescent to explore values independently and the development of a personal value system. The laissez-faire approach involves little or no guidance and can lead to confusion and conflict. Through modeling, children learn that which is of high or low value by observing parents, peers, and significant others. The moralizing mode of value transmission teaches a complete value system and allows little opportunity for the weighing of different values. Responsible choice encourages children to explore competing values and to weigh their consequences while support and guidance are offered.

The nurse is to administer a potassium supplement to the client. The nurse does not check the potassium level prior to administering the medication and later finds that the potassium level was at a critical high. What principle has this nurse violated? o Beneficence o Autonomy o Nonmaleficence o Fidelity

o Nonmaleficence · Nonmaleficence is the duty to do no harm to the client. For instance, if a nurse fails to check a prescription for an unusually high dose of insulin and administers it, the nurse has violated the principle of nonmaleficence. Beneficence is the duty to do good for the clients assigned to the nurse's care. The nurse has a duty to remove wrist restraints whenever possible (removing a harm) and to help the client regain independence (promoting and doing good). Fidelity is the duty to maintain commitments of professional obligations and responsibilities. Autonomy refers to a client's right to self-determination or the freedom to make choices without opposition.

A client with chronic obstructive pulmonary disease presents with respiratory acidosis and hypoxemia. The client tells the nurse that they don't want to be placed on a ventilator. What action should the nurse take? o Notify the physician immediately to have the physician determine client competency. o Consult the palliative care group to direct care for the client. o Have the client sign a do-not-resuscitate (DNR) form. o Determine whether the client's family was consulted about this decision.

o Notify the physician immediately to have the physician determine client competency. · Three requirements are necessary for informed decision-making: the decision must be given voluntarily; the client making the decision must have the capacity and competence to understand; and the client must be given adequate information to make the decision. In light of the client's respiratory acidosis and hypoxemia, the client might not be competent to make this decision. The physician should be notified immediately so the physician can determine client competency. The physician, not the nurse, is responsible for discussing the implications of a DNR order with the client. The Patient's Bill of Rights entitles the client to make decisions about the care plan, including the right to refuse recommended treatment. The client's family may oppose the client's decision. Consulting the palliative care group isn't appropriate at this time and must be initiated by a physician order.

The nurse is caring for a combative and confused client with a fractured hip who is trying to get out of bed. What actions are appropriate for the nurse to take? Choose all that apply. o Obtain a physician's order to restrain the client. o Ask a family member to sit with the client. o Leave the client momentarily to obtain soft restraints from the clean supply room. o Apply restraints and leave the room to notify a physician. o Give the client a sedative.

o Obtain a physician's order to restrain the client. o Ask a family member to sit with the client. · It is mandatory in most settings to have a physician's order before restraining a client. Before restraints are used, other strategies, such as asking family members to sit with the client or using a specially trained sitter, should be tried. The Joint Commission (formerly the Joint Commission on Accreditation of Healthcare Organizations, or JCAHO) and the Centers for Medicare and Medicaid Services (CMS) have designated standards for the use of restraints. A client should never be left alone while the nurse summons assistance. All staff members require annual instruction on the use of restraints, and the nurse should be familiar with the facility's policy. This makes all other answers incorrect.

While administering a medication via a syringe, a client sharply moves and the nurse accidentally encounters a needlestick. What is the priority nursing action? o Document the injury. o Report the needlestick to the nurse manager. o Obtain the client's blood to be tested for HIV and HBV. o Request counseling on the potential for infection.

o Report the needlestick to the nurse manager. · Upon encountering a needlestick, the nurse's priority action is to report the injury. Other actions can take place after the injury has been reported.

To ensure ethical nursing care when dealing with genetic and genomic information, which principle would the nurse integrate as the foundation for all nursing care? o Justice o Respect for people o Veracity o Fidelity

o Respect for people · Although fidelity, veracity, and justice are ethical principles that may be involved with ethical issues surrounding genetic and genomic information, respect for people is the ethical principle underlying all nursing care.

Following a long history of multiple visits to community clinics and emergency departments, a client has been diagnosed with hypochondriasis. During this current visit to the emergency department, the client has just been informed that diagnostic testing and assessment reveal no severe illness. Despite this, the client persists in verbalizing physical complaints. How should the nurse respond to this? o Feign an assessment of the client in order to calm the client's anxiety. o Have a different member of the care team present the test and assessment results to the client. o Facilitate a repeat of the previous diagnostic testing in order to appease the client. o Set limits with the client about the complaints.

o Set limits with the client about the complaints. · If a client with the diagnosis of hypochondriasis has been told that the client has no life-threatening or severe illnesses, but the client continues to verbalize clinical symptoms, limit-setting is used. A "false" assessment is unethical, and repeating diagnostic testing reinforces the client's behavior. Having diagnostic results presented by another member of the care team is unlikely to eliminate the client's concerns.

The nurse provides care to a verbally unresponsive client diagnosed with terminal cancer. The client's family refuses palliation on religious grounds. The nurse experiences great anxiety and distress when caring for the client due to the level of suffering perceived. What action should the nurse take? o Discuss the plan of care with the client's healthcare provider. o Accept that this is the client's and family's wish. o Speak to the charge nurse about the nurse's moral conflict. o File a complaint with the facility's client advocate.

o Speak to the charge nurse about the nurse's moral conflict. · The nurse is experiencing moral conflict and needs to discuss this with the charge nurse. The nurse is not able to simply ignore the feelings being experienced and accept the family's wish; if that were the case, there would be no issue to begin with. If there is an ethical breach related to the client's care, the nurse's first action is to speak with the healthcare team, not with the client advocate. In this case, a true ethical issue is not established. The goal is to strike a balance between the religious freedom of the client and family, the moral autonomy of the nurse, and the delivery of care that exhibits non-malfeasance. After consulting with the charge nurse, it may be decided to excuse the nurse from working with the client on moral grounds, or that the ethical committee needs to be consulted.

The nurse is caring for a 17-year-old girl in the terminal phase of osteosarcoma. Which action demonstrates integration of the recommendations of the American Academy of Pediatrics (AAP) Committee on Bioethics? o Allowing the child to listen during discussions of the care plan. o Explaining the prognosis using accepted clinical terminology. o Telling the child exactly what to expect of further treatments. o Encouraging the child to support the wishes of her parents.

o Telling the child exactly what to expect of further treatments. · The committee recommends telling the child exactly what to expect of further treatments and procedures, explaining the prognosis in a developmentally appropriate way to ensure the child's understanding, and endeavoring to gain the child's candid opinion of the proposed care plan. It also recommends that decision-making for older children and adolescents should include the assent of the child or adolescent.

Which electrocardiogram (ECG) characteristic is usually seen when a client's serum potassium level is low? o T wave o P wave o QT interval o U wave

o U wave · The U wave is an ECG waveform characteristic that may reflect Purkinje fiber repolarization. It is usually seen when a client's serum potassium level is low. The T wave is an ECG characteristic reflecting repolarization of the ventricles. It may become tall or "peaked" if a client's serum potassium level is high. The P wave is an ECG characteristic reflecting conduction of an electrical impulse through the atria. The QT interval is an ECG characteristic reflecting the time from ventricular depolarization to repolarization.

A nurse is caring for a client who was raped at gunpoint. The client does not want any photos taken of the injuries. The client also does not want the police to be informed about the incident even though state laws require reporting life-threatening injuries. Which intervention should the nurse perform to document and report the findings of the case? o Obtain photos to substantiate the client's case in a court of law. o Respect the client's opinion and avoid informing the police. o Document only descriptions of medical interventions taken. o Use direct quotes and specific language.

o Use direct quotes and specific language. · The nurse should use direct quotes and specific language as much as possible when documenting. The nurse should not obtain photos of the client without informed consent. The nurse should, however, document the refusal of the client to be photographed. Documentation must include details as to the frequency and severity of abuse and the location, extent, and outcome of injuries, not just a description of the interventions taken. The nurse is required by law to inform the police of any injuries that involve knives, firearms, or other deadly weapons or that present life-threatening injuries.

When an ethical decision is made based on the reasoning of the "greatest good for the greatest number," what theory is the nurse following? o Moral-justification theory o Utilitarian theory o Formalist theory o Deontological theory

o Utilitarian theory · One classic theory in ethics is teleologic theory or consequentialism, which focuses on the ends or consequences of actions. The best-known form of this theory, utilitarianism, is based on the concept of "the greatest good for the greatest number." Another theory in ethics is the deontological or formalist theory, which argues that ethical standards or principles exist independently of the ends or consequences.

One classic theory in ethics is teleologic theory or consequentialism, which focuses on the ends or consequences of actions. The nurse knows that the best known form of this theory is o Utilitarianism o Beneficence o Double effect o Formalist theory

o Utilitarianism · One classic theory in ethics is teleologic theory or consequentialism, which focuses on the ends or consequences of actions. The best-known form of this theory, utilitarianism, is based on the concept of "the greatest good for the greatest number." Beneficence and double effect are examples of common ethical principles. The formalist theory is another theory in ethics which argues that ethical standards or principles exist independently of the ends or consequences.

What is the term for the beliefs held by the individual about what matters? o Ethics o Values o Bioethics o Morals

o Values · Values are ideals and beliefs held by an individual or group about what matters; values act as a standard to guide one's behavior. Ethics are moral principles and values that guide the behavior of honorable people. A moral is a standard for right and wrong. Bioethics is related to ethical questions surrounding life and death, as well as questions and concerns regarding quality of life as it relates to advanced technology.

What are standards for decision-making that endure for a significant time in one's life? o Beliefs o Roles o Ethics o Values

o Values · Values are standards for decision-making that endure for a significant time in one's life. Beliefs are more likely to be changeable and are not necessarily a long-term foundation for decision-making. Ethics is a systematic study of principles of right and wrong conduct, virtue and vice, and good and evil as they relate to conduct and human flourishing. A role is a set of responsibilities one takes on in relationship to others.

A nurse has completed 4 hours of an 8-hour shift on a medical-surgical unit when the nurse receives a phone call from the nursing supervisor. The nursing supervisor informs the nurse that the nurse needs to give report to the other two nurses on the medical-surgical unit and immediately report to the telemetry unit to assist with staff needs on that unit. The nurse informs the supervisor that the nurse has been busy with the current client assignment and feels this will overwhelm the nurses on the medical-surgical unit. The supervisor informs the nurse that the need is greater on the telemetry unit. This is an example of which type of ethical problem? o allocation of scarce nursing resources o conflicts concerning new technology o deception o advocacy in a market-driven economy

o allocation of scarce nursing resources · This is a clear example of ethical issues surrounding the allocation of scarce nursing resources and involves distributing nursing care.

A client with autoimmune thrombocytopenia and a platelet count of 8,000/μl develops epistaxis and melena. Treatment with corticosteroids and immunoglobulins has been unsuccessful, and the physician recommends a splenectomy. The client states, "I don't need surgery — this will go away on its own." In considering her response to the client, the nurse must depend on the ethical principle of: o justice. o autonomy. o advocacy. o beneficence.

o autonomy. · Autonomy is the right of the individual to make his own decisions. In this case, the client is capable of making his own decision and the nurse should support his autonomy. Beneficence, promoting and doing good, and justice (being fair) aren't the principles that directly relate to the situation. Advocacy is the nurse's role in supporting the principle of autonomy.

The nurse is initiating isolation precautions for a client who has chronic Clostridium difficile infection. What should the nurse be sure to include with these precautions? o include a N95 respirator mask for health care staff entering the room o remind others to use a mask when caring for this client o recognize that this type of infection requires droplet precautions o be sure that there are gloves of various sizes and gowns for use

o be sure that there are gloves of various sizes and gowns for use · All health care workers and visitors should don a gown and gloves prior to entering the client's room. These bacteria are not transmitted by droplet. An N95 respirator mask is not required for this client.

Providing milieu therapy is an example of the use of which ethical principle? o fidelity o veracity o beneficence o autonomy

o beneficence · When using the ethical principle of beneficence, a health care provider uses knowledge of science and incorporates the art of caring to develop an environment in which individuals achieve their maximal health care potential, as in milieu therapy. Veracity is the duty to tell the truth. Fidelity is faithfulness to obligations and duties. According to the principle of autonomy, each person has the fundamental right of self-determination.

Which written medical instruction explicitly states that no action should be taken to revive a client if he or she experiences cardiac arrest? o do-not-resuscitate order o living will o advance directive o durable power of attorney

o do-not-resuscitate order · Do-not-resuscitate (DNR) orders involve a written medical order for end-of-life instructions. If a DNR order is written, the client wishes to have no resuscitative action taken if he or she experiences a cardiac arrest. An advance directive provides the means for clients to communicate their wishes regarding life-sustaining treatment and other medical care, so that their significant others will know what decisions the clients desire. It is not a medical order. A living will is a type of advance directive; it is not a medical order. A durable power of attorney is a legal document that appoints a person to act as an agent for another person. A DPOA for healthcare appoints a person to make medical decisions for a client who is incapacitated and unable to make decisions for himself or herself.

A client with severe acute respiratory syndrome privately informs the nurse of a desire not to be placed on a ventilator if the condition worsens. The client's partner and children have repeatedly expressed their desire that every measure be taken for the client. The most appropriate intervention by the nurse would be to: o encourage the client to consider a living will or power of attorney. o ask the physician to discuss the client's prognosis with the client and the family. o arrange a conference to discuss the matter with all involved. o assure the client that all possible measures will be taken.

o encourage the client to consider a living will or power of attorney. · The nurse is obligated to act as the client's advocate. A living will or power of attorney would clearly define the client's wishes. The nurse should not discuss the issue with the client's family unless the client gives permission. Assuring the family and client that all possible measures will be taken opposes the client's wishes and does not demonstrate client advocacy.

A nurse finds that a colleague is intoxicated while on duty. What appropriate action would the nurse take? o inform the nursing supervisor o ask another nurse if the colleague appears intoxicated o ask the colleague if he or she is intoxicated o advise the colleague to go home

o inform the nursing supervisor · When a colleague is intoxicated while on duty, the nurse should immediately inform the nursing supervisor, who may take necessary action. It would be an irresponsible action if the nurse tells the colleague to go home. Confronting the colleague by asking if he or she is intoxicated may result in denials and an attempt to avoid any repercussions. The nurse should not involve other staff members to protect the privacy of the colleague. Only the nursing supervisor needs to be made aware of the situation.

A nurse discusses the HIV-positive status of a client with other colleagues. The client can sue the nurse for which violation? o defamation of character o invasion of privacy o professional negligence o false imprisonment

o invasion of privacy · The client can sue the nurse for invasion of privacy. Disclosing confidential information to an inappropriate third party subjects the nurse to liability for invasion of privacy, even if the information is true. Defamation of character includes false communication that results in injury to a person's reputation. Prevention of movement or unjustified retention of a person without consent may be false imprisonment. Negligence is an act of omission or commission.

The foundation for decisions about resource allocation throughout a society or group is based on the ethical principle of: o autonomy. o veracity. o justice. o confidentiality.

o justice. · Justice is the foundation for decisions about ethical resource allocation throughout a society or group, because resources must be allocated fairly and equitably. The other ethical principles listed, veracity, autonomy, and confidentiality, have no direct application to resource allocation within society.

During the 1800s, Pinel believed that the cure for mental illness was ... o placement outside of the community. o arrest and confinement. o moral treatment. o use of chains.

o moral treatment. · During the 1800s, the cure for mental illness was believed to be moral treatment, defined as kindness, compassion, and a pleasant environment. Philippe Pinel was one of the first physicians who began using moral treatment in France.

During the 1800s, Pinel believed that the cure for mental illness was ... o placement outside of the community. o moral treatment. o arrest and confinement. o use of chains.

o moral treatment. · During the 1800s, the cure for mental illness was believed to be moral treatment, defined as kindness, compassion, and a pleasant environment. Philippe Pinel was one of the first physicians who began using moral treatment in France.

When assessing if a procedural risk to a client is justified, the ethical principle underlying the dilemma is known as what? o pro-choice o informed consent o nonmaleficence o self-determination

o nonmaleficence · Nonmaleficence is the principle of creating no harm. It refers to preventing or minimizing harm to an individual. The other options do not represent the situation presented in the question.

A group of nursing students are reviewing current nursing codes of ethics. Such a code is important in the nursing profession because: o nurses are highly vulnerable to criminal and civil prosecution in the course of their work o nurses are responsible for carrying out actions that have been ordered by other individuals o nurses interact with clients and families from diverse cultural and religious backgrounds o nursing practice involves numerous interactions between laws and individual values

o nursing practice involves numerous interactions between laws and individual values · A code of ethics is necessary to guide nurses' conduct, especially with regard to the interaction between laws and individual values. Diversity and legal liability do not provide the main justification for a code of ethics, though each is often a relevant consideration. The fact that nurses often carry out the orders of others is not the justification for a code of ethics.

Which part of Freud's personality structure is associated with ethics, standards, and self-criticism? o ego o id o superego o libido

o superego · The superego is the part of the personality structure associated with ethics, standards, and self-criticism. The id is formed by unconscious desires, primitive instincts, and unstructured drives, including sexual desires and aggressive tendencies that arise from the body. The ego consists of the sum of certain mental mechanisms, such as perception, memory, and motor control, as well as specific defense mechanisms. The energy or psychic drive associated with the sexual instinct, or libido, resides in the id.

When a client is admitted to the hospital, admissions personnel are required to determine whether the client has a document indicating advanced directives. If so, a copy is made for the client's medical record. The advanced directive document indicates: o that an attorney has verified the living will papers. o that the client refuses to have resuscitation measures or any life-prolonging care. o that the client has made wishes for terminal care known. o that the client assigned a relative to act on the client's behalf.

o that the client has made wishes for terminal care known. · It is important to determine whether the client has advanced directives, which indicate the client's wishes regarding future care should the client become unable to communicate them. Advanced directives may be documented in a living will or a durable power of attorney for health care document. A copy should be placed in the client's hospital record. Simply having an advanced directive only means that the client has expressed some wishes regarding terminal care, not necessarily that an attorney has verified the document, that the client has refused to have resuscitation measures or any life-prolonging care, or that the client has assigned a relative to act on the client's behalf, although any or all of these could be true.

A nurse who works on a palliative care unit has participated in several clinical scenarios that have required the application of ethics. Ethics is best defined as o the laws that govern acceptable and unacceptable behavior o moral values are considered to be universal o the relationship between law and culture o the principles that determine whether an act is right or wrong

o the principles that determine whether an act is right or wrong · Ethics involves moral or philosophical principles that direct actions as being either right or wrong. Laws are often rooted in ethics but the two terms are not synonymous. Similarly, morals and values are closely associated with ethics but these do not constitute the definition of ethics. Ethics are not universally agreed upon, as many different applications exist.

A public health nurse is involved in planning a community outreach program for a large assisted living community. Due to the aging population within the community, the program will offer hypertension screening and management. This decision is based on which principle? o veracity o nonmaleficence o autonomy o utilitarianism

o utilitarianism · Teleology, also known as utilitarianism, is ethical decision-making process based on final outcomes and what is best for the most people. The choice that benefits many people justifies harm that may come to a few. The nurse did not display veracity, nonmaleficence, or autonomy in this scenario.

A client who experienced a cesarean birth rates pain as a 9 on a 10-point scale. The nurse medicates the client for pain. This is an example of the nurse practicing which ethical principle? o Fidelity o Beneficence o Nonmaleficence o Justice

o Beneficence · Beneficence means doing or promoting good. The treatment of the client's pain is the nurse's act of doing good. Justice refers to treating clients fairly and impartially. Fidelity refers to keeping one's promises and acting faithfully. Nonmaleficence refers to preventing or not causing harm to the client.

A nursing student is making notes that include client data on a clipboard. Which statement by the nursing instructor is most appropriate? o "Clipboards with client data should not leave the unit." o "You can get an electronic printout of client lab data to take with you." o "Be sure to write down specific information for your clinical paperwork." o "Be sure to put the client's name and room number on all paperwork."

o "Clipboards with client data should not leave the unit." · HIPAA has created several changes that protect client confidentiality and affect the workplace. One such change is that the names of clients on charts can no longer be visible to the public, and clipboards must obscure identifiable names of clients and private information about them. Therefore, writing down clinical information, taking the data off the unit, and including client identifiers are inappropriate.

A charge nurse has implemented staff education on nursing values. The nurse would determine that further education is required when which statement(s) are overheard? Select all that apply. o "If you are going to have extramarital sex, please protect yourself by using a condom." o "I can't believe the client is giving that precious baby up for adoption." o "Smoking has been shown to be a risk for many illnesses, including heart disease and cancer." o "If that was my mother, I sure wouldn't agree to a no-code." o "The gonorrhea test was positive. That's what the client gets for sleeping around."

o "I can't believe the client is giving that precious baby up for adoption." o "If that was my mother, I sure wouldn't agree to a no-code." o "The gonorrhea test was positive. That's what the client gets for sleeping around." · Being judgmental, as in the options about adoption, gonorrhea, and code-status, does not reflect the values desired in a nurse. Professional nurses do not assume that their personal values are more correct than those of their clients. Nurses are obligated, however, to provide health information such as recommending condom use and smoking cessation.

Nurse A is having difficulty logging into the automated medication-dispensing system, and asks Nurse B to log in momentarily so that Nurse A is not delayed in administering client medications. What is Nurse B's appropriate response? o "I am giving you my password so you can log in." o "I will log in so that you can proceed with medication delivery." o "I will get the hospital's information system's phone number for you." o "I can log in and give the medications for you."

o "I will get the hospital's information system's phone number for you." · Passwords and logins should never be shared with anyone else, nor should a nurse use his or her own password or login information to allow another individual to access the automated medication-dispensing system. Nurse B will not log in and give the medications, but rather will provide a solution by offering contact information for information systems to Nurse A so that he or she can work through their login issue.

A client of a homecare nurse gives the nurse an envelope with a small amount of money in it, stating, "It's a tip for the good care you give me." Which statement would be the most appropriate response from the nurse? o "I'm not allowed to accept gifts of money, but if you wanted to give me something else, that would be acceptable." o "Although I can't accept this money, you could just let my supervisor know you're pleased with my work!" o "I'm grateful that you're satisfied with the care you're receiving, but I can't accept any form of gift." o "Thank you, this is very generous of you. I'm really humbled by this token of your appreciation."

o "I'm grateful that you're satisfied with the care you're receiving, but I can't accept any form of gift." · Because the nurse is in a position of power, it would be an abuse of power to accept the gift; specifically, it would be considered financial abuse. This is also true of non-monetary gifts. Asking the client to speak to the manager on the nurse's behalf is unprofessional.

A nurse is determining whether or not informed consent has been obtained from the family of a child who is going to have abdominal surgery. Which statement by the family would lead the nurse to suspect that informed consent is lacking? o "Although there are risks involved, our son needs the surgery to cure the problem." o "We are amazed that he'll be up and walking around the day after surgery." o "We had to sign the form right away so the surgery could get scheduled." o "He might miss some school afterwards, but he'll be feeling much better."

o "We had to sign the form right away so the surgery could get scheduled." · The statement about signing the form right away suggests that the family was coerced into agreeing to the surgery without being fully informed about the risks and benefits. The key ethical issues related to informed consent for treatment have similarities to those required for research participation: Consent must be voluntary and based upon shared information about the risks and benefits of the treatment. Furthermore, the parent must understand the information and be cognitively and mentally competent to make the decision. The statements about risks, activity limitations, and postoperative care indicate that information was shared with them and that they understood it.

A client recruited to participate in a clinical trial to treat non-Hodgkin's lymphoma voices concerns about the adverse effects of the treatment preventing the completion of the trial. What will the nurse tell the client to protect the principle of autonomy? o "Adverse effects will be managed with medication." o "There are risks and benefits associated with trial participation." o "You may withdraw at any time." o "Participation in the trial may benefit others."

o "You may withdraw at any time." · Although the nurse must tell the client how adverse effects will be treated and the risks and benefits associated with participating in the trial, the most crucial element in protecting the ethical principle of autonomy is to inform the client that they may withdraw at any time without punitive consequences. Telling the client how participation in the trial will benefit others takes the focus from the client and does not support autonomy.

What is provided in the Code of Ethics for Nurses of the American Nurses Association (ANA)? o A description of case studies featuring ethical dilemmas o Information about what to do when confronted with an ethical dilemma o A guideline for nurses regarding ethical conduct o Pathways to follow for each of the ethical principles

o A guideline for nurses regarding ethical conduct · The ANA's Code of Ethics for Nurses guides ethical decision-making.

Before administering an immunization to their child, the nurse asks parents to take which priority action? o Provide the child's immunization record. o Sign a consent form. o Assist in restraining the child. o Reassure the child.

o Sign a consent form. · Parents must sign a consent form before immunization of the child after receiving full information about the vaccines, their importance, and their administration. Reassuring the child and assisting in restraining are both important but are not the priority. Having the child's immunization record with them allows this record to be updated; otherwise, a full record should be given to the parent.

The nurse in the emergency department is caring for an 8-year-old child brought in by ambulance after being struck by a car while riding a bike. No family member is present or reachable and the child needs emergency surgery. How would the nurse expect the need for informed consent to be addressed? Select all that apply. o An emergency social worker would sign the informed consent as a parental proxy. o A surgeon may choose to forgo informed consent if it is a lifesaving measure. o Surgery may be done without an informed consent by the family in this situation. o Surgery would have to wait until one of the child's parents could provide consent. o The ethics committee would have to convene to approve surgery in this situation.

o A surgeon may choose to forgo informed consent if it is a lifesaving measure. o Surgery may be done without an informed consent by the family in this situation. · It may be necessary for the surgeon to operate as a lifesaving measure without the client or family's informed consent. Every effort must be made to contact the client's family. Neither a social worker nor a member of the ethics committee may sign an informed consent. In this situation, the surgery is considered lifesaving, so a delay would be unacceptable.

A nurse is caring for a client who is a practicing Jehovah's Witness. The health care provider prescribes two units of packed red blood cells based on the client's low hemoglobin and hematocrit levels. The nurse states to the health care provider that it is unethical to go against the client's beliefs even though the client's blood counts are very low. What is the best description of the nurse's intentions? o Observing institutional policies o Being legally responsible o Siding with the client over the health care provider o Acting in the client's best interest

o Acting in the client's best interest · Nurses' ethical obligations include acting in the best interest of their clients, not only as individual practitioners, but also as members of the nursing profession, the health care team, and the community at large. In this case, the nurse is aware of the client's belief that it is wrong to receive a blood transfusion and acts to protect the right of the client to self-determination in care, thus acting in the client's best interest. The nurse is not simply taking the client's side against the health care provider, as the nurse is observing the ethical principle of autonomy. Although the nurse may be also observing institutional policy and being legally responsible in this act, the nurse's primary commitment is to the client.

A client with stage IV heart failure documents in an advance directive that no ventilatory support should be provided. What should the nurse do when the client begins experiencing severe dyspnea? o Administer oxygen, morphine, and a bronchodilator for comfort. o Ask the healthcare provider to prescribe bilevel positive airway pressure (BIPAP). o Coach the client to take slow deep breaths. o Ask the client's family to consent to ventilator placement.

o Administer oxygen, morphine, and a bronchodilator for comfort. · An advance directive identifies a client's wishes in the event that a life-threatening illness or injury occurs. The client's comfort should be paramount and the nurse should respect the client's wishes. Morphine, oxygen, and bronchodilators can relieve dyspnea and make the client more comfortable, which will make breathing easier. The client will need more than coaching to take slow deep breaths. It is a violation of the client's advance directive to ask the family to consent to a ventilator. BIPAP is used to treat sleep apnea and not acute shortness of breath.

Which role of the nurse-client relationship is being exhibited when the nurse informs the client and then supports him or her in whatever decision he or she makes? o Parent surrogate o Advocate o Teacher o Caregiver

o Advocate · In the advocate role, the nurse informs the client and then supports him or her in whatever decision he or she makes. The primary caregiving role in mental health settings is the implementation of the therapeutic relationship to build trust and explore feelings. In the teacher role, the nurse instructs the client about the medication regimen. In the role of the parent surrogate, the nurse may be tempted to assume a parental role.

A nurse is working within the managed care delivery model. Which is true regarding managed care? o Their values are not reflected in the decision making. o All plans have the same values underlying the delivery of care. o All systems reflect the values of efficiency and effectiveness. o There are no conflicts between cost-effectiveness and respectful care.

o All systems reflect the values of efficiency and effectiveness. · All systems in the managed care delivery model reflect the values of efficiency and effectiveness. Different plans may have different values underlying the delivery of care. However, they all reflect the business plan values of efficiency and effectiveness. Their values are reflected in the decision making and the policy development of the organization. Value conflicts between cost-effectiveness and respectful care may be seen.

Which organization has established standards that help the nurse determine which clinical actions fall under the scope of nursing practice? o National Council of State Boards of Nursing o International Council of Nurses o National League for Nursing o American Nurses Association

o American Nurses Association · The American Nurses Association (ANA) has established standards of clinical nursing practice that define the specific and unique activities of nurses. Standards serve as protection for the nurse, the client, and the institution where health care is given. The other organizations provide valuable services that differ from those of ANA. The National Council of State Boards of Nursing have the nurse practice acts which define state specific guidelines for nurses to practice. The National League for Nursing is a national organization for faculty nurses and leaders in nurse education. The International Council of Nurses is a federation of more than 130 national nurses associations.

Which is the most important reason for psychiatric nurses to understand law, legislation, and legal processes that relate to professional nursing practice? o Because only by lobbying can psychiatric nurses have an impact on the delivery of services on a national level. o Because these activities are included in the Nursing Code of Ethics. o Because doing so gives the nurse the ability to provide quality care that will safeguard the rights and safety of clients. o Because doing so gives the nurse guidelines by which to use seclusion and restraint appropriately, when needed.

o Because doing so gives the nurse the ability to provide quality care that will safeguard the rights and safety of clients. · Nurses practice under the Code of Ethics and the Scope of Practice. Nurses must learn to value, respect, and develop knowledge about laws, legislation, and the legal processes that regulate, impede, and facilitate professional nursing practice.

A nurse is caring for an alert and oriented client with end-stage breast cancer who has been unable to eat for 4 days due to intractable nausea. The nurse calls the health care provider (HCP) and obtains an order for an NG tube placement per the spouse's request. When the nurse brings the equipment to the room, the client turns to the spouse and states "I just told you that I don't want any tubes or feedings!" Using the ANA Code of Ethics for Nurses, place the nursing actions in sequential order (rank actions from first to last). All options must be used. o Explore other options that would be acceptable to the client. o Notify the HCP of the client's wishes and decision. o Verify that the client understands the purpose of the NG tube. o Document the client's wishes in the medical record. o Educate the husband on the wife's right to refusal of care. o Apologize for not asking the client prior to calling the HCP.

o Apologize for not asking the client prior to calling the HCP. o Verify that the client understands the purpose of the NG tube. o Explore other options that would be acceptable to the client. o Educate the husband on the wife's right to refusal of care. o Document the client's wishes in the medical record. o Notify the HCP of the client's wishes and decision. · The ANA Code of Ethics for Nurses directs that a nurse promote, advocate for, and strive to protect the health, safety, and rights of the patient. The client has the right to refusal and is able to make an informed decision. The nurse should first apologize, then make sure the client has enough information to make an informed decision, including exploring other options, then educate the husband about the wife's right to refuse treatment. This should be followed by documentation of the client's wishes and notification of the HCP.

The client has become confused and attempts to climb out of bed. What interventions will the nurse provide prior to applying restraints? o Place all four side rails of the bed in the upright position. o Call the health care provider to prescribe sedation for the patient. o Place the client in a chair at the nurses' station with a sheet tied around the client's waist. o Arrange a schedule for staff to sit with the client.

o Arrange a schedule for staff to sit with the client. · The nurse should arrange a schedule for the staff to sit with the client. Calling for sedation is not the first step with caring for a client with confusion. The chair with a sheet and the side rails are restraints. The use of restraints (including physical and pharmacologic measures) is another issue with ethical overtones because of the limits on a person's autonomy when restraints are used. It is important to weigh carefully the risks of limiting autonomy and increasing the risks of injury by using restraints against the risks of injury if not using restraints, which have been documented as resulting in physical harm and death. The ANA advocates that restraints only be used when no other viable option is available. The Joint Commission and the Centers for Medicare and Medicaid Services (CMS) have designated standards for the use of restraints.

A nurse volunteers to serve on the hospital ethics committee. Which action should the nurse expect to take as a member of the ethics committee? o Convince the family to choose a specific course of action. o Assist in decision making based on the client's best interests. o Decide the care for a client who is unable to voice an opinion. o Present options about the type of care.

o Assist in decision making based on the client's best interests. · One reason an ethics committee convenes is when a client is unable to make an end-of-life decision and the family cannot come to a consensus. In this case, the committee members are there to advocate for the best interest of the client and to promote shared decision making between the client (or surrogates, if the client is legally incapacitated) and the clinicians. The committee would not convince, decide, or present options about the type of care. This is not the role of an ethics committee.

The nurse is caring for a critically ill client who informs the nurse that there is a conflict between the client's spiritual beliefs and a proposed health option. What is the nurse's role in this situation? o Inform the client's healthcare provider of the client's concerns. o Provide examples of ways clients handle spiritual and care planning conflicts. o Assist the client in obtaining information to make an informed decision. o Encourage the client to pray for clarity on the matter and offer support.

o Assist the client in obtaining information to make an informed decision. · The nurse's role in resolving conflicts between spiritual beliefs and treatments is to assist the client in obtaining the information needed to make an informed decision and to support the client's decision making. Telling the client to pray does not assist the client in the decision-making process. The healthcare provider does not manage client decisions when it comes to spiritual beliefs. Offering examples of other clients does not help this client.

The use of patient restraints limits which ethical principle? o Justice o Autonomy o Trust o Beneficence

o Autonomy · It is important to weigh carefully the risk of limiting a client's autonomy and increasing the risk of injury by using restraints against the risk of not using restraints. Beneficence refers to an act of goodness, justice in nursing often refers to bioethics and means giving to others what is due, finally trust is building a relationship based on reliability and truths. The patient's autonomy is limited with the use of restraints.

A nurse is providing care to a client with end-stage cancer. After weighing the alternatives, the client decides not to participate in a clinical trial offered and is requesting no further treatment. The nurse advocates for the client's decision based on the understanding that the client has the right to self-determination, interpreting the client's decision as reflecting which ethical principle? o Autonomy o Beneficence o Justice o Fidelity

o Autonomy · When respecting autonomy, the nurse supports the client's right to make decisions with informed consent. When promoting the client's well-being, the nurse acts in the best interests of the client. Advocacy is linked to the belief that making choices about health is a fundamental human right that promotes the individual's dignity and well-being. Beneficence is reflected by doing good and promoting what will benefit the client. Justice involves treating each client fairly. Fidelity involves being faithful and keeping promises.

Which ethical principle refers to the obligation to do good? o Beneficence o Veracity o Fidelity o Nonmaleficence

o Beneficence · Beneficence is the duty to do good and the active promotion of benevolent acts. Fidelity refers to the duty to be faithful to one's commitments. Veracity is the obligation to tell the truth. Nonmaleficence is the duty not to inflict, as well as to prevent and remove, harm; however, there will be times when the action may cause "harm" to promote the "good" such as chemotherapy being used to treat and eliminate cancer, but it causes various adverse effects while trying to reach the ultimate goal..

A client with persistent depression is considering electroconvulsive therapy (ECT). The nurse has seen ECT be effective in other cases. When the client expresses fear and doubt about undergoing ECT, the nurse tries to talk the client into it, because the nurse truly believes it will help the client. Which two ethical concepts are in conflict? o Justice and autonomy o Beneficence and fidelity o Fidelity and paternalism o Beneficence and autonomy

o Beneficence and autonomy · Beneficence and autonomy are in conflict. Beneficence is practicing with the intent to do good; however, professionals define how to do good, which may override the wishes and self-determination of the client. Autonomy is the client's right to make decisions for himself or herself. Justice refers to fairness; that is, treating all people fairly and equally without regard for social and economic status, race, sex, marital status, religion, ethnicity and cultural beliefs. Fidelity is the nurse's faithfulness to duties, obligations, and promises.

Which theory of ethics prioritizes the nurse's relationship with clients and the nurse's character in the practice of ethical nursing? o Principle-based ethics o Utilitarianism o Deontology o Care-based ethics

o Care-based ethics · Central to the care-based approach to ethics is the nurse's relationships with clients and the nurse's "being," or character and identity. Deontology is the study of the nature of duty and obligation. Utilitarianism is the doctrine that actions are right if they are useful or for the benefit of a majority. Principle-based ethics prioritizes goals and principles that exist beyond the particularities of the nurse-client relationship.

The nurse is providing care to a woman who has just given birth to a healthy term neonate. The woman's partner arrives and asks about the neonate's status. Which action by the nurse would be appropriate? o Ask the partner for identification first before sharing any information. o Check the medical record for written client approval with whom to share information. o Answer the partner's questions honestly and without hesitation. o Tell the partner that no information can be shared with him or her at this time.

o Check the medical record for written client approval with whom to share information. · In maternal and newborn health care, information is shared only with the client, legal partner, parents, legal guardians, or individuals as established in writing by the client or the child's parents. This law promotes the security and privacy of health care and health information for all clients. Therefore, the nurse needs to check the medical record for written documentation that allows the partner to have this information. Any other action would be inappropriate.

A client is undergoing genetic testing and counseling. What is the responsibility of the nurses during this process? o Explain the results of the testing to the client. o Clarify values and goals for the couple. o Share with the client the nurse's opinion when asked. o Notify insurance companies of test results.

o Clarify values and goals for the couple. · Nurses can help clients clarify values and goals, assess understanding of information, protect clients' rights, and support a client's decisions. Notifying insurance companies of test results and giving the client a personal opinion, even when asked, is unethical on the part of the nurse. The nurse can ensure the client understands the results, but the nurse should not explain the results of the test to the client; this is the primary health care provider's role.

A client involved in a motor vehicle collision has awakened from a coma and asks for his wife, who was killed in the same accident. The family does not want the client to know at this time that his wife was killed. The family wants all nursing staff to tell the client that the wife was air lifted to another hospital, has a severe head injury, and is in the ICU. Because the American Nurses Association Code of Ethics requires the nurse to preserve integrity, but the nurse wants to follow the family's instructions, the nurse faces an ethical dilemma. The steps of ethical analysis can assist the nurse with decision making. Select from the list below all the steps that are correct. o Collect information. o Decide and evaluate the decision. o List the alternatives. o Recognize the ethical, legal, and professional dimensions involved. o Coordinate an ethics committee.

o Collect information. o Decide and evaluate the decision. o List the alternatives. o Recognize the ethical, legal, and professional dimensions involved. · Assess the ethical/moral situations of the problem. This step entails recognition of the ethical, legal, and professional dimensions involved. The second step is to collect information. The next step is to list the alternatives. Compare alternatives with applicable ethical principles and professional code of ethics. The last step is to decide and evaluate the decision. The steps of an ethical analysis are for individual nurses to be able to process an ethical dilemma on their own.

The nurse is preparing to give medications to a client with high blood pressure. The prescription indicates that the client is to have the combination drug dextroamphetamine saccharate-amphetamine aspartate monohydrate-dextroamphetamine sulfate-amphetamine sulfate 40 mg by mouth twice daily. What is the appropriate nursing action? o Administer the drug as prescribed. o Contact the health care provider for clarification of the prescription. o Assume that the provider meant to prescribe propranolol. o Ask another nurse to verify the prescription.

o Contact the health care provider for clarification of the prescription. · Before administering the medication, the nurse should immediately contact the health care provider to verify the prescription; no one else can verify the prescription. The combination drug dextroamphetamine saccharate-amphetamine aspartate monohydrate-dextroamphetamine sulfate-amphetamine sulfate and the drug propranolol are medications that have look-alike and sound-alike properties, but are very different in indication and dosage.

The nursing staff is divided over withdrawing care from a competent, chronically ill client. The nurse-manager would take which step to meet the needs of the staff? o Assign only nurses who agree with the client's plan of care. o Arrange a meeting with the client's family and nursing staff. o Encourage the staff to talk to the client about their concerns. o Contact the institutional ethics committee for strategies.

o Contact the institutional ethics committee for strategies. · The institutional ethics committee can help the staff develop strategies to resolve their ethical dilemma. The Patient's Bill of Rights states that the client (not the family) has the right to make decisions about the care plan and to refuse recommended treatment. Arranging a meeting with the client's family is inappropriate, whether or not they are in agreement with the client's wishes. Assigning only nurses that agree with the client's wishes is not a reasonable staffing option. Talking to the client about their concerns is inappropriate as it takes the focus away from the client.

There are many ethical issues in the care of clients with HIV or HIV/AIDS. What is an ethical issue healthcare providers deal with when caring for clients with HIV/AIDS? o Sharing the diagnosis with a support group o Disclosure of the client's condition o Caring for a client who can kill other people o Caring for a client with an infectious terminal disease

o Disclosure of the client's condition · Despite HIV-specific confidentiality laws, clients infected with AIDS fear that disclosure of their condition will affect employment, health insurance coverage, and even housing. Since healthcare providers do not share a client's diagnosis with a support group, option A is incorrect. Caring for a client with an infectious terminal illness that can be transmitted to other people is a concern for healthcare providers but it is not an ethical issue.

A nurse overhears a second nurse making plans to meet a hospitalized client for a drink after the client has been discharged. Which is the best action for the first nurse to take? o Tell the client not to meet the nurse socially. o Encourage the interaction with the client after discharge. o Discuss the conversation directly with the other nurse. o Report the conversation to the nurse manager.

o Discuss the conversation directly with the other nurse. · Planning to meet a client for a social event while the client is still hospitalized could blur the boundaries of the therapeutic relationship. This could result in an unhealthy outcome for the client. The nurse should take the second nurse aside and point out that the behavior is inappropriate and not in the client's best interest. The other options do not demonstrate behavior that is consistent with the therapeutic nurse-client relationship.

A nurse observes another nurse making social plans with a client and disclosing information of a personal nature. What would the observing nurse do in this situation? o Let coworkers know what is going on to have witnesses. o Report the observation to the nurse manager. o Find out whether the nurse meets with other clients socially as well. o Discuss the observation directly with the nurse.

o Discuss the observation directly with the nurse. · Planning to meet a client socially and disclosing personal information could blur the boundaries of the therapeutic relationship, which may result in an unhealthy outcome for the client. The observing nurse should take the nurse aside and point out that this behavior is inappropriate and not in the client's best interest. Sharing the observation with coworkers is gossiping and does not address the issue. As a professional, the nurse has an obligation to help educate the other nurse. It is possible a reminder about professional boundaries will resolve the issue without requiring involvement of the nurse manager. Regardless if the nurse has met with other clients or not, this particular instance should be dealt with by directly discussing the concerns with the nurse.

In some cases, the act of providing nursing care in unexpected situations is covered by the Good Samaritan laws. Which nursing action would most likely be covered by these laws? o Emergency care for a choking victim in a restaurant o Any emergency care given when consent is obtained o A negligent act performed in an emergency situation o Medical advice given to a neighbor regarding a child's rash

o Emergency care for a choking victim in a restaurant · Good Samaritan laws are designed to protect health practitioners when they give aid to people in emergency situations in which the practitioner is off duty, such as providing emergency care to a choking victim in a restaurant. The other examples listed are not situations covered by the Good Samaritan law.

A nurse cultivates dispositions that enable practicing nursing in a manner in which he or she believes in. This nurse is displaying what essential element of ethical agency? o Ethical sensibility o Ethical character o Ethical valuing o Ethical responsiveness

o Ethical character · Ethical character is the development or cultivation of virtues such as humility, courage, and integrity to name just a few. The development or cultivation of these virtues allows the nurse to practice in a manner in which the nurse believes in.

A hospice nurse is caring for a client with terminal cancer. The family would like the client to continue aggressive therapy to treat the cancer, but the client has voiced to the nurse that no further pursuit of treatment is desired. The nurse speaks to the family about the client's wishes, condition, and terminal state. This action is most likely derived from which nursing obligation? o Nursing education principles o Ethical principles o Advanced practice licensure guidelines o Legal responsibilities

o Ethical principles · Ethics is a systematic study of principles of right and wrong conduct, virtue and vice, and good and evil as they relate to conduct and human flourishing. In this case, the nurse is practicing the ethical principle of autonomy, which is the client's right to self-determination related to choices about care. The nurse's action is not primarily motivated by legal responsibility, nursing education principles, or advanced practice licensure guidelines.

Healthcare providers use a problem-solving approach for ethical dilemmas. Which is the last step of the ethical decision-making model? o Survey other healthcare professionals to see if they agree with the decision. o Keep detailed documentation of the entire decision-making process. o Evaluate the decision in terms of effects and results. o Follow through on the decision that has been made.

o Evaluate the decision in terms of effects and results. · Evaluating the decision is the very important last step. Making the decision and following through on it are important, but they are not the last step. Surveying other healthcare professionals is not part of the ethical decision-making model. Detailed documentation is important in regard to many professional duties, but it is not the last step of the ethical decision-making model.

Earlier in the shift, the nurse promised to help a client acquire some paper and a pen and draft a letter to a family member later in the day. The nurse became increasingly busy during the shift but has now taken some time to assist the client in this way. What ethical principle has the nurse best exemplified? o Nonmaleficence o Beneficence o Veracity o Fidelity

o Fidelity · Fidelity involves keeping promises. Veracity is truth-telling while beneficence is doing good and nonmaleficence is avoiding harm.

A client rings a call bell to request pain medication. Upon performing the pain assessment, the nurse informs the client that the nurse will return with the pain medication. The nurse's promise to return with the pain medication is an example of which principle of bioethics? o fidelity o nonmaleficence o autonomy o justice

o fidelity · Fidelity is keeping one's promises and never abandoning a client entrusted to care without first providing for the client's needs. Autonomy respects the rights of clients or their surrogates to make healthcare decisions. Nonmaleficence is avoiding causing harm. Justice involves giving each his or her due and acting fairly.

A nursing student is initiating a relationship with an assigned client. After meeting and spending approximately 20 minutes talking with the client, the student makes arrangements to visit again after lunch. After lunch, fellow classmates invite the student to go to the gym with them and a group of clients to play volleyball. The student starts to go with them but then remembers the promise to meet with the client. The student decides to forgo volleyball and talk with the client. The student's decision reflects which ethical principle? o Autonomy o Veracity o Beneficence o Fidelity

o Fidelity · Fidelity is faithfulness to obligations and duties. It is keeping promises. Fidelity is important in establishing trusting relationships. With autonomy, each person has the fundamental right of self-determination. According to the principle of beneficence, a health care provider uses knowledge of science and incorporates the art of caring to develop an environment in which individuals achieve their maximal health care potential. Veracity is the duty to tell the truth.

A nurse states to the client that the nurse will keep the client free of pain. However, the client's family wishes to try a treatment to prolong the client's life that may necessitate withholding pain medication. This factor will cause an ethical dilemma for the nurse in relation to which ethical principle? o Fidelity o Autonomy o Veracity o Justice

o Fidelity · In this case, the nurse's promise to keep the client pain-free is the best example of the principle of fidelity means being faithful to one's commitments and promises. Veracity refers to telling the truth. Justice refers to treating clients fairly and equitably. Autonomy refers to respecting the client's right to self-determination.

A nurse observes individuals who are volunteering in a community free health clinic. Which of the following stages of Erikson's Psychosocial Development theory describes these individuals? o Intimacy vs isolation o Generativity vs stagnation o Ego Integrity vs despair o Identity vs role confusion

o Generativity vs stagnation · With Erikson's Generativity vs Stagnation the individual desires to make a difference in his or her community. Volunteering at the community free health clinic is an example of this. Ego Integrity vs despair is a time of reminiscence of their life. Intimacy vs isolation is seen in young adults who are meshing their identities with those of their friends and making commitments. Identity vs role confusion is the transition from childhood to adulthood.

A nurse is searching the Internet for information about theorists associated with moral development. The search reveals several theorists. Which result would the nurse identify as being associated with women and morality? o Gilligan o Kohlberg o Piaget o Fowler

o Gilligan · Gilligan (1977; 1982) originally worked with Kohlberg. As she listened to women discuss their own real-life moral conflicts, she recognized that there was a conception of morality from the female viewpoint that was not represented in Kohlberg's work. Gilligan's theory views girls and women as developing a morality of response and care, and boys and men as developing a morality of justice. Piaget developed a theory of cognitive development. Fowler postulated a developmental theory of the spiritual identity of humans.

A nurse overhears a colleague tell a client that based on the genetic testing results she should terminate the pregnancy. Which action is most appropriate for the nurse to take? o Tell the unlicensed assistive personnel o Contact the health care provider. o Call the client later and apologize. o Immediately stop the nurse.

o Immediately stop the nurse. · The nurse should interrupt the nurse and remind him or her that it is important not to impose personal values onto the client.

A client with metastatic brain cancer is admitted to the oncology floor. What action will the admitting nurse take regarding an advanced directive for this client? o Respect the individuals' moral rights. o Decide on a treatment plan if the client cannot. o Advise the client to refuse medical treatment if unable to make health care decisions. o Inform the client or legal guardian of the right to execute an advance directive.

o Inform the client or legal guardian of the right to execute an advance directive. · All clients have a right to execute an advance directive. The admitting nurse would ensure that the client is aware of that right. The facility's ethics committee can decide on a treatment plan if the client is unable and a health care power of attorney has not been appointed. Facility employees are not required by law to respect an individual's moral rights; however, the health care professional should respect the client's individual rights as part of professional responsibility. While a client may refuse medical treatment via an advanced directive, the nurse would not advise this.

A client who tested positive for human immunodeficiency virus (HIV) and has pancreatitis is admitted to the medical unit. The nurse director from another unit comes into the medical unit nurses' station and begins reading the client's chart. The staff nurse questions the director about reading the client's chart. The director states that the client is a neighbor's son. What action should the nurse take? o Remind the nurse director not to share the client's medical information with anyone because of the client's HIV status. o Inform the nurse director reading the chart is a violation of the client's right to privacy and ask the nurse director to return the chart. o Report the incident to the medical director and document the nurse director's actions. o Remind the nurse director that permission from the medical director must be provided before access can be granted.

o Inform the nurse director reading the chart is a violation of the client's right to privacy and ask the nurse director to return the chart. · Personal health information may not be used for purposes not related to health care. The nurse director found reading the chart is not providing health care to the client and therefore does not require access to the chart. The nurse should confront the nurse director and request the return of the client's chart. The director should not have access to this client's health care information regardless of the client's HIV status. If the nurse director does not comply with the nurse's request, the nurse should report the incident to the nurse manager, so the infraction can be reported through the proper channels. The staff nurse should not report the incident to the medical director. Asking the nurse director about permission from a medical director to read the chart does not protect client confidentiality.

When making ethical decisions about caring for preschoolers, a nurse should remember to o make decisions that will prevent legal trouble. o do what the nurse would do for a close child or loved ones. o provide beneficial care and avoid harming the child. o be sure to do what the physician says.

o provide beneficial care and avoid harming the child. · Nurses must provide beneficial care and avoid harming all clients. A nurse shouldn't base any decision solely on the desire to prevent legal trouble, on the nurses own feelings for loved ones, or what the physician says.

A nurse reports to the charge nurse that a client medication due at 9 am was omitted. Which principle is the nurse demonstrating? o Altruism o Autonomy o Integrity o Social justice

o Integrity · The principle of integrity is based on the honesty of a nurse according to professional standards. In this instance, the nurse reported the occurrence of the missed medication to the charge nurse. The definition of altruism is concern for others; it can best be explained by a nurse concerned about how a client will care for self after discharge. Social justice is a concept of fair and just relations between the individual and society and is related to wealth and distribution of goods in a society. Autonomy is the right to self-determination or acting independently and making decisions.

The nurse beginning practice would like to access the standards for ethical practice. Which organization should the nurse research for these standards? o Department of Health Professions o World Health Organization o Canadian Hospital Association o International Council of Nurses

o International Council of Nurses · The American Nurses Association Code of Ethics for Nurses is the main source of the ethical principles that nurses must follow, and this is developed by the International Council of Nurses. The Canadian Hospital Association has not developed standards of ethical practice. Individual state boards of nursing through the department of health professions have laws and regulations that guide practice but not ethical and moral principles. The World Health Organization is a large organization that does not determine ethical practice for nurses.

An HIV-positive client discovers that the client's name is published in a research report on HIV care prepared by the client's nurse. The client is hurt and files a lawsuit against the nurse. Which offense has the nurse committed? o Defamation of character o Unintentional tort o Invasion of privacy o Negligence of duty

o Invasion of privacy · The nurse has committed the tort of invasion of privacy. Personal names and identities should be concealed or obliterated in case studies or research work. Invasion of privacy is a type of intentional tort. Defamation is an act in which untrue information harms a person's reputation and is therefore not applicable here. Negligence is the harm that results because a person did not act reasonably.

Which of the following is a true statement about critical thinking according to Alfaro-LeFevre (2006)? o It makes judgments based on conjecture. o It is based on the medical model. o It considers only the client's needs. o It is guided by professional standards and codes of ethics.

o It is guided by professional standards and codes of ethics. · Critical thinking is guided by professional standards and codes of ethics. It is based on principles of the nursing process and scientific methods. Critical thinking makes judgments based on evidence rather than conjecture. It considers client, family, and community needs.

A client comes to the emergency department with severe depression and suicidal ideation. Staff members determine that the client does not have adequate insurance to cover inpatient psychiatric services at their facility, so they discharge the client with some prescriptions for medication. Which principle is being ignored by discharging this client? o Justice o None of the above o Veracity o Autonomy

o Justice · Justice is the duty to treat all clients fairly. It can become an ethical issue in mental health when a segment of the population does not have access to care, as in this case, in which access to inpatient care is warranted but denied.

A nurse is working with young adults and assessing their moral development. The nurse relates which of the following theorists who developed the Theory of Moral Development? o Lawrence Kohlberg o Erik Erikson o Sigmund Freud o Robert Havighurst

o Lawrence Kohlberg · Lawrence Kohlberg developed the Theory of Moral Development. Erik Erikson developed the Theory of Psychosocial Development. Robert Havighurst described learned behaviors as developmental tasks that occur through various times in a human's life. Sigmund Freud developed the theory of Psychoanalytic Development.

Which action most clearly demonstrates a nurse's commitment to social justice? o Answering a client's questions about care clearly and accurately o Documenting client care in a timely, honest, and thorough manner o Ensuring that a hospital client's diet is culturally acceptable o Lobbying for an expansion of healthcare resources and benefits to those in poverty

o Lobbying for an expansion of healthcare resources and benefits to those in poverty · Social justice is a professional value that encompasses efforts to promote universal access to health care, such as the expansion of publicly funded programs like Medicare. Culturally competent care is a reflection of human dignity. Answering clients' questions and documenting care accurately are expressions of the value of integrity.

The parent of an 11-year-old girl with an inoperable brain tumor confides to the nurse that her daughter's physician is "pushing them" to convince their daughter to participate in a controversial treatment that has a high risk for side effects. She further states that she told him twice that they were not interested. What would be the nurse's best response to this situation? o Tell the doctor that the procedure is not in the best interests of this girl and ask him to respect the rights of the family. o Reassure the family that the doctor is acting in their best interests and they should reconsider the treatment. o Meet with the physician and disclose the concerns of the family; refer the case to the institutional ethics committee if not resolved. o Tell the parents to state in writing that they will not participate in the treatment and give the letter to their physician.

o Meet with the physician and disclose the concerns of the family; refer the case to the institutional ethics committee if not resolved. · When a nurse believes the physician has unduly coerced parents in their treatment decision, the nurse would be obliged to intervene and disclose any concerns. Such intentional or unintentional action would violate ethical principles of conduct.

A nurse in a psychiatric care unit finds that a client with psychosis has become violent and has struck another client in the unit. What action should the nurse take in this case? o Restrain the client, as they are harmful to the other clients. o Do not restrain the client, as it is equivalent to false imprisonment. o Inform the physician and complete a comprehensive assessment. o Do not restrain the client, as it is equivalent to battery.

o Restrain the client, as they are harmful to the other clients. · The nurse should restrain the client because they are potentially harmful to other clients in the psychiatric care unit. Restraints should be used as a last resort and their use should be justified. Unnecessary restraining can lead to allegations of false imprisonment and battery; both are not applicable in this case, however. The nurse should inform the physician about the client but sometimes it may not be logical to wait for orders to restrain a violent client.

A nurse is caring for a client with multiple sclerosis. The client informs the nurse that a lawyer is coming to prepare a living will and requests the nurse to sign as witness. Which action should the nurse take? o Note that the nurse caring for the client cannot be a witness. o State that the physician will be a witness. o Inform the physician about the living will. o Arrange for another colleague to sign as a witness.

o Note that the nurse caring for the client cannot be a witness. · A living will is an instructive form of an advance directive. It is a written document that identifies a person's preferences regarding medical interventions to use in a terminal condition, irreversible coma, or persistent vegetative state with no hope of recovery. Employees of the healthcare facility should not sign as witnesses; therefore, the nurse cannot sign as witness. Refusing a client may not be a good communication method; instead, the nurse could politely indicate the reason for declining. Calling for a physician or asking another colleague to sign is an inappropriate action.

The healthy adult client is given an opioid prior to a surgical procedure. The nurse is completing the chart and notices the consent form was not signed by the client. Which of the following should the nurse do first? o Immediately have the client sign the consent form. o Notify the physician of the oversight. o Ask the client if he still wants to proceed with the procedure. o Have the client's family member sign the consent form.

o Notify the physician of the oversight. · Do not administer any medications that might alter judgment or perception before the client signs the consent form because many drugs commonly administered as preoperative medications, such as opioids or barbiturates, can alter cognitive abilities and invalidate informed consent.

A family member of a resident in a long-term care facility reports to the nurse that her mother's diamond ring is missing. Another resident reported a day earlier that a twenty-dollar bill was missing from his/her night table. What should the nurse do in this situation? o Pass the information on to the doctor and the next shift staff. o Remind the residents and family members not to leave valuables unattended. o Notify the supervisor and call the police. o Report the incidents to the facility's lawyer.

o Notify the supervisor and call the police. · The supervisor should be made aware of the situation and the police should be called to investigate the potential theft. The other answers do not advocate for the clients and their families. It is the responsibility of the nurse to take action because the nurse was the person to receive the information. This is known as due diligence.

A nurse is caring for a client with a terminal illness. The client asks the nurse to help him end his own life to alleviate his suffering and that of his family. When responding to the client, the nurse integrates knowledge of which of the following? o Nurses may administer medications prescribed by physicians to hasten end of life. o Participating in assisted suicide violates the Code of Ethics for Nurses. o A client has the right to make independent decisions about the timing of his or her death. o Most states have enacted laws that allow for physician-assisted suicide.

o Participating in assisted suicide violates the Code of Ethics for Nurses. · The American Nurses Association Position Statement on Assisted Suicide acknowledges the complexity of the assisted suicide debate but clearly states that nursing participation in assisted suicide is a violation of the Code for Nurses. Legally, nurses are not allowed to administer medications even if prescribed by a physician if that medication will hasten the client's end of life. Proponents of physician-assisted suicide argue that terminally ill people should have a legally sanctioned right to make independent decisions about the value of their lives and the timing and circumstances of their deaths. However, this is not the case at the present time. Two states have enacted legislation for physician-assisted suicide. These laws provide access to physician-assisted suicide by terminally ill clients under very controlled circumstances.

A nurse caring for older adults in a long-term care facility is teaching a novice nurse characteristic behaviors of older adults. Which statement is not considered ageism? o Most older adults are ill and institutionalized. o Old age begins at age 65. o Intelligence declines with age. o Personality is not changed by chronologic aging.

o Personality is not changed by chronologic aging. · Ageism is a form of prejudice, like racism, in which older adults are stereotyped by characteristics found in only a few members of their group. Fundamental to ageism is the view that older people are different and will remain different; therefore, they do not experience the same desires, needs, and concerns as other adults. The statement not considered ageism would be that personality is not changed by chronologic aging. Most older adults are not ill and institutionalized. Intelligence does not decline with age. Old age does not begin at age 65.

Which nursing situations are examples of the care-based approach to ethics? Select all that apply. o Documenting data in the electronic medical record o Providing a back rub to a client on bed rest o Holding the hand of a dying client o Involving the parent in the bed bath of a child o Taking a week's vacation from work

o Providing a back rub to a client on bed rest o Holding the hand of a dying client o Involving the parent in the bed bath of a child · Nursing interventions related to care and the client demonstrate a care-based approach. Holding the hand of a dying client, providing a back rub, and involving the parents are all examples of a care-based approach. Documenting care provided in the electronic health record is not care-based but part of everyday responsibilities of the nurse. Taking a week of vacation is a personal intervention and not related to the care of a client.

A nurse working in the operating room is assigned to the suite where therapeutic abortions are to be performed throughout the day. The nurse feels that participation in these procedures conflicts with personal religious beliefs. What should the nurse do after notifying the operating room supervisor? o Continue working in the suite because that is the assignment for the shift. o Complete a work refusal form and leave the surgical suite immediately. o Remain in the operating room suite until another nurse arrives to take that assignment. o Contact the local right-to-life association and inform them of the procedures.

o Remain in the operating room suite until another nurse arrives to take that assignment. · If nursing care is requested that is contrary to the nurse's personal values, the nurse must provide appropriate care until alternative care arrangements are in place to meet the client's needs. The other options are not correct, and if the nurse left the suite, it could result in the accusation that the nurse abandoned the client. The nurse should be aware of potential conflicts before accepting an assignment.

A nurse, while off-duty, tells the physiotherapist that a client who was admitted to the nursing unit contracted AIDS due to exposure to sex workers at the age of 18. The client discovers that the nurse has revealed the information to the physiotherapist. With what legal action could the nurse be charged? o Libel o Slander o Negligence o Malpractice

o Slander · The nurse can be charged with slander, which is a verbal attack on a person's character. Libel pertains to damaging written statements read by others. Both libel and slander are considered defamation of character-an intentional tort in which one party makes derogatory remarks about another that diminish the other party's reputation. To be found guilty of slander or libel, the statement must be proved false. Negligence and malpractice pertain to actions which are committed or omitted, thereby causing physical harm to a client.

An older couple approaches the nurse's station seeking information about their hospitalized grandchild. What should be the nurse's response? o Assume "busy-looking" behaviors and move away. o Tell the couple to seek information directly from the child's parents. o Answer the grandparents' questions in a less public place. o Ask to see identification from the two individuals.

o Tell the couple to seek information directly from the child's parents. · To protect the child's privacy, information can be shared only with legal guardians, parents, or those identified in writing. "Looking busy" only temporarily avoids confronting the privacy rights.

Which is the best definition of ethics? o The adherence to formal personal values o The formal, systematic study of moral beliefs o The adherence to informal personal values o The informal, systematic study of moral beliefs

o The formal, systematic study of moral beliefs · Ethics is a formal, systematic inquiry into principles of right and wrong conduct, of virtue and vice, and of good and evil as they relate to human conduct and human flourishing. "Morals" usually refers to personal or communal standards of right and wrong.

A 50-year-old client who has recently been diagnosed with a chronic degenerative illness has announced to the nurse the intention to commit suicide in order to prevent future suffering. Which fact should underlie the nurse's response to this client? o The nurse is ethically obliged to inform law enforcement. o The nurse must refer the client to a physician who is authorized to assist the client with a suicide. o The nurse is obliged to protect the client from self-harm. o The nurse is required to document the client's wishes and begin to facilitate an assisted suicide.

o The nurse is obliged to protect the client from self-harm. · While the nurse is not obliged to inform law enforcement, he or she is ethically obligated to protect the client from self-harm. Participation or referral for assisted suicide has not been recognized as an acceptable component of nursing practice.

Nursing students are reviewing legislation about the use of genetic information. The students demonstrate a need for additional review when they identify which of the following as being prohibited by the Genetic Information Nondiscrimination Act (GINA)? o Health insurers from charging higher rates for people at risk for a genetic condition o Use of genetic information to establish insurance eligibility o Employers from using genetic information to make a decision about promotions o Health insurers from denying coverage to a healthy person at risk for a genetic condition

o Use of genetic information to establish insurance eligibility · The Health Insurance Portability and Accountability Act (HIPAA) prohibits the use of genetics information to establish insurance eligibility. The GINA prohibits insurers from denying coverage to a healthy person or charging higher insurance rates based on a person's genetic predisposition to a disease. It also prevents employers from using a person's genetic and genomic information to make decisions about hiring, job placement, promotion, or firing.

A hospital board of directors decides to close a pediatric burn treatment center (BTC) that annually admits 50 patients and to open a treatment center for terminally ill AIDS patients (with an expected annual admission of 200). This decision means that the nearest BTC for children is now 300 miles away. What example of ethical reasoning is this decision consistent with? o Obligation or duty o "The means justifies the end" o A formalist approach o Utilitarianism

o Utilitarianism · One classic theory in ethics is teleologic theory or consequentialism, which focuses on the ends or consequences of actions. The best-known form of this theory, utilitarianism, is based on the concept of "the greatest good for the greatest number." Another theory in ethics is the deontological or formalist theory, which argues that ethical standards or principles exist independently of the ends or consequences. Beneficence is the obligation or duty to do good and the active promotion of benevolent acts (e.g., goodness, kindness, charity).

A client with bipolar disorder has been following the prescribed medication regimen. The client indicates to the nurse a desire to stop the medication now that the client is feeling better. The nurse tells the client that most likely the client will have to remain on the medication for life to keep the condition under control. The nurse is practicing which principle? o Veracity o Fidelity o Justice o Autonomy

o Veracity · Veracity is the duty to tell the truth. In this case, the client wants to hear that the client can stop medication, but the nurse is honest and tells the client that the client will need to continue it to stay healthy.

The nurse has prepared a syringe containing morphine to give to a client. The client refuses the drug, stating that pain is under control. What procedure does the nurse follow to properly dispose of the medication? o Two nurses must physically return the medication to the pharmacy for disposal, and both nurses are required to sign the controlled substance return log. o Waste the medication with a nurse witness present, and the nurse witness must countersign the control record. o Discard the controlled medication and document the amount, date, and time on the control record, then sign it. o A second and third nurse must witness the discarding of the controlled medication, and all three nurses must sign the control record.

o Waste the medication with a nurse witness present, and the nurse witness must countersign the control record. · Opioids like morphine must strictly be accounted for because they are controlled substances. If the client refuses the medication, the nurse must waste that medication with a nurse witness present and document this process. Two witnesses are not needed, only one. It is not appropriate to waste the medication without the nurse witness present or return the controlled substance to the pharmacy.

A nurse is presenting a workshop for a group of young women. Part of the workshop focuses on the moral development of women. The nurse integrates information about Gilligan's theory into the presentation, describing the various levels that a woman goes through as morality develops. When describing level 1, the nurse would focus on which area? o Move from selfishness to responsibility o Acceptance by others o Woman's own needs o Adoption of social values

o Woman's own needs · In level 1, the focus is on the girl's or woman's own needs. Should and would are the same. The transition that follows this level is characterized by the move from selfishness to responsibility—a move that integrates the responsibility to care for oneself with the desire to care for others. In level 2, moral judgment is based on shared norms and expectations, and societal values are adopted. Acceptance by others becomes critical, and the ability to protect and care for others becomes the defining characteristic of female goodness.

The nurse is teaching a client about the importance of adhering to a medication regimen. The client does not believe that it is important. The nurse is communicating which ethical principle? o justice o beneficence o paternalism o veracity

o beneficence · According to the principle of beneficence, the health care provider uses knowledge of science and incorporates the art of caring to develop an environment in which individuals achieve their maximum health care potential. Justice is the duty to treat all fairly, distributing the risk and benefits equally. Veracity is the duty to tell the truth. Paternalism is the belief that knowledge and education authorize professionals to make decisions for the good of the client.

Which theory of ethics most highly prioritizes the nurse's relationship with clients and the nurse's character in the practice of ethical nursing? o deontology o utilitarianism o care-based ethics o principle-based ethics

o care-based ethics · Central to the care-based approach to ethics is the nurse's relationships with clients and the nurse's "being," or character and identity. Deontology concerns duty and obligation. Utilitarianism prioritizes the greatest happiness of the greatest number of people. Principle-based ethics is a framework that focuses on autonomy, non-maleficence/beneficence, and justice. Each prioritizes goals and principles outside the particularities of the nurse-client relationship.

A client being treated for complications of chronic obstructive pulmonary disease needs to be intubated. The client has previously discussed their wish to not be intubated with the client's partner of 5 years, whom the client has designated as healthcare power of attorney. The client's children want their parent to be intubated. A nurse caring for this client knows that o the client's partner is responsible for national legislation regarding surrogate decision makers. o clients commonly confer healthcare power of attorney on someone who shares their personal values and beliefs. o healthcare providers must honor the children's wishes to avoid a lawsuit. o the children's biological relationship with their parent supersedes the partner's wishes.

o clients commonly confer healthcare power of attorney on someone who shares their personal values and beliefs. · The healthcare power of attorney is someone who can make decisions when the client can't. Clients tend to select individuals who share their personal values and beliefs as their healthcare power of attorney. Family members and designated surrogates don't always agree; state laws regarding surrogate decision makers may differ. The legal rights of a healthcare power of attorney in regards to healthcare decisions supersede those of family members. The law designates the healthcare power of attorney as the person to make decision; violating this designation could result in a lawsuit.

Several days this week, a nurse takes time after work to read to a visually impaired client who has no family close by. This behavior demonstrates that ethical values: o are well thought out. o are consistent. o take priority over other values. o concern the treatment of others.

o concern the treatment of others. · Ethical values are rules or principles a person uses to make decisions about right and wrong. These values are consistent, they take priority over other values, they concern the treatment of others (as shown by this nurse taking personal time to show kindness to her client), and they are well thought out.

A nurse is acting inappropriately and has an odor of alcohol. This behavior breaches the principle of: o fidelity. o autonomy. o beneficence. o ethical conduct.

o ethical conduct. · Ethical conduct dictates that a nurse act in a professional, safe, competent manner. Drinking alcohol while at work and behaving inappropriately breaches this principle. Beneficence is acting for the good of the client. Fidelity is keeping promises. Autonomy is respecting one's right to self-determination and making decisions. Although the nurse's intoxication will likely lead to breaches in the principles of beneficence, fidelity, and autonomy, there is no evidence of such breaches in these specific principles in this case.

Several times, family members have asked a nurse to share personal prescriptions when they were in need of pain medication or antibiotics. Which type of rules or standards should govern the nurse's moral decision? o civil law o common law o ethics o administrative law

o ethics · Although all of the options may affect the decision, moral decisions are guided by ethics (moral principles and values that guide the behavior of honorable people). Ethical standards dictate the rightness or wrongness of human behavior. Laws are written rules for conduct and actions. They are binding for all citizens and ensure the protection of rights.

A client is refusing to take the prescribed oral medication. Which measure by the nurse can be used to get the client to take the medication? Select all that apply. o having a family member give the medication o explaining the purpose of the medication to the client o crushing the medication and hiding it in apple sauce o asking the client the reason for not taking the medication o suggesting a liquid form of the medication instead of a pill

o explaining the purpose of the medication to the client o asking the client the reason for not taking the medication o suggesting a liquid form of the medication instead of a pill · The correct answers provide an alternative solution for the client and provide the client an opportunity to consent to taking the medication in another form, neither of which would be considered abuse. Providing health education regarding the medications to ensure the client has all the information needed to make an informed consent would be appropriate. Hiding medication or disguising it in food knowing that the client has refused the medication would be considered abuse. The client has the right to refuse care, including medication, and a family member should not be placed in a position of having to give the medication.

The nurse is obtaining informed consent from a client. To adhere to ethical and legal standards, the nurse must ensure that the informed consent consists of what? Select all that apply. o freedom from coercion o discussion of pertinent information o caregiver preference and opinion o verification from next of kin o the client's agreement to the plan of care

o freedom from coercion o discussion of pertinent information o the client's agreement to the plan of care · Discussion of pertinent information, the client's agreement to the plan of care, and freedom from coercion are important factors in informed consent. Caregiver preference and opinion could be perceived as coercion. Informed consent does not require verification from next of kin.

According to the Health Insurance Portability and Accountability Act (HIPAA) passed in 1996, clients: o can be punished for violating guidelines. o need to obtain legal representation to update their health records. o have the right to copy their health records. o are required to obtain health record information through their insurance company.

o have the right to copy their health records. · HIPAA affords clients the right to see and copy their health records, update their health records, and get a list of disclosures that a health care institution has made for the purposes of treatment, payment, and health care operations. Clients have the right to request a restriction on certain uses or disclosures and choose how to receive this health information. HIPAA includes punishments for anyone caught violating client privacy, but these punishments are not directed at the client because HIPAA was implemented to protect the privacy of an individual's health information.

A nurse discusses the HIV-positive status of a client with other colleagues. The client can sue the nurse for which violation? o invasion of privacy o professional negligence o defamation of character o false imprisonment

o invasion of privacy · The client can sue the nurse for invasion of privacy. Disclosing confidential information to an inappropriate third party subjects the nurse to liability for invasion of privacy, even if the information is true. Defamation of character includes false communication that results in injury to a person's reputation. Prevention of movement or unjustified retention of a person without consent may be false imprisonment. Negligence is an act of omission or commission.

While riding in the elevator, a nurse discusses the HIV-positive status of a client with other colleagues. The nurse's action reflects: o false imprisonment. o defamation of character. o professional negligence. o invasion of privacy.

o invasion of privacy. · The nurse's action reflects an invasion of the client's privacy. Disclosing confidential information to an unauthorized third party subjects the nurse to liability for invasion of privacy, even if the information is true. Defamation of character includes false communication that results in injury to a person's reputation. Negligence is an act of omission or commission. Prevention of movement or unjustified retention of a person without consent may be false imprisonment.

A nurse may use self-disclosure with a client if: o the client asks directly about the nurse's experience. o the nurse has experienced the same situation as the client. o it helps the client to talk more easily. o it achieves a specific therapeutic goal.

o it achieves a specific therapeutic goal. · Self-disclosure (making personal statements about oneself) can be a useful nursing tool. However, a nurse should use self-disclosure judiciously and with a specific therapeutic purpose in mind. The nurse should listen closely to the client and remember that the experiences of different people are sometimes similar but never identical. Using too many self-disclosures is unethical and can shift the focus from the client to the nurse. Self-disclosure that distracts the client from treatment issues doesn't benefit the client and may alienate the client from the nurse.

Which ethical principles become an issue in mental health when a segment of a population does not have access to health care? o justice o nonmaleficence o veracity o fidelity

o justice · Justice is the duty to treat all fairly, distributing the risk and benefits equally. Justice becomes an issue in mental health when a segment of a population does not have access to health care. Basic good should be distributed so the least advantaged members of society benefit. Nonmaleficence, fidelity, and veracity are not being used in this situation.

Ethical distress is: o knowing the correct action but being unable to perform it due to constraints. o having trouble deciding which of two conflicting ethical principles to follow. o being aware of the principles of right and wrong. o supporting the rights of a client during hospitalization.

o knowing the correct action but being unable to perform it due to constraints. · Knowing the correct action but being unable to perform it due to constraints is the definition of moral or ethical distress. Being aware of knowing what is right and wrong is the first step of having an ethical conscience. Supporting a client during a hospitalization is an ethical principle of doing what is right (beneficence) and the focus of client-centered care. An ethical dilemma is a situation in which a person has difficulty deciding which of two or more conflicting ethical principles to follow.

A client is brought to the emergency department by an adult child, who states, "I am unable to care for my parent anymore. Although I would like to, financially and physically I cannot do it anymore." What ethical problem is the adult child experiencing? o dissatisfaction o ethical dilemma o moral distress o ethical uncertainty

o moral distress · Moral distress is when someone wants to do the right thing but is not able. The adult child brings the parent to the emergency department to maintain the parent's safety, although the child needs to take care of the parent. Dilemma is a situation in which a difficult choice has to be made between two or more alternatives, especially equally undesirable ones. The client is not experiencing a dilemma. Uncertainty means a feeling of not knowing what will happen. The adult child is certain that he or she cannot care for the parent. Dissatisfaction implies a sense of dislike for, or unhappiness in, one's surroundings. Although the adult child is unhappy, this is not the primary ethical problem.

After teaching a class to a group of nursing students about reporting infectious diseases to the Centers for Disease Control and Prevention, the instructor determines a need for additional discussion when the students identify which infection as being reportable: o pertussis o pinworm o Lyme disease o gonorrhea

o pinworm · Pinworm infections are not required to be reported. Gonorrhea, Lyme disease, and pertussis are all reportable infectious diseases.

A client with colorectal cancer has been presented with her treatment options but wishes to defer any decisions to her uncle, who acts in the role of a family patriarch within the client's culture. What best protects the client's right to self-determination? o holding a family meeting and encouraging the client to speak on her own behalf o revisiting the decision when the uncle is not present at the bedside o teaching the client about her right to autonomy o respecting the client's desire to have the uncle make choices on her behalf

o respecting the client's desire to have the uncle make choices on her behalf · The right to self-determination (autonomy) means that decision-making should never be forced on anyone. The client has the autonomous right to defer her decision making to another individual if she freely chooses to do so.

When a client is admitted to the hospital, admissions personnel are required to determine whether the client has a document indicating advanced directives. If so, a copy is made for the client's medical record. The advanced directive document indicates: o that an attorney has verified the living will papers. o that the client assigned a relative to act on the client's behalf. o that the client has made wishes for terminal care known. o that the client refuses to have resuscitation measures or any life-prolonging care.

o that the client has made wishes for terminal care known. · It is important to determine whether the client has advanced directives, which indicate the client's wishes regarding future care should the client become unable to communicate them. Advanced directives may be documented in a living will or a durable power of attorney for health care document. A copy should be placed in the client's hospital record. Simply having an advanced directive only means that the client has expressed some wishes regarding terminal care, not necessarily that an attorney has verified the document, that the client has refused to have resuscitation measures or any life-prolonging care, or that the client has assigned a relative to act on the client's behalf, although any or all of these could be true.

A nurse is discussing principles in healthcare ethics with the nursing students. Which would be an appropriate example of nonmaleficence? o to administer pain medications to a client in pain o to protect clients from a chemically impaired practitioner o to provide emotional support to clients who are anxious o to perform dressing changes to promote wound healing

o to protect clients from a chemically impaired practitioner · Protecting clients from a chemically impaired practitioner is an appropriate example of nonmaleficence, which means to avoid doing harm, to remove from harm, and to prevent harm. Performing dressing changes to promote wound healing, providing emotional support to anxious clients, and administering pain medications to a client having pain are examples of beneficence, which means doing or promoting good.

A day-shift nurse gives a client an injection of pain medication. The nurse forgets to document the injection on the medication administration record (MAR). The day-shift nurse tells the evening-shift nurse that she gave the client 4 mg of morphine at 2 p.m. for postoperative pain but didn't document the injection. The evening-shift nurse puts the day-shift nurse's initials and the date and time the dose was administered in the appropriate area of the MAR. The evening-shift nurse's action is considered to be which type of documentation error? o omission o improper correction o unauthorized entry o late entry

o unauthorized entry · This action is an unauthorized entry. A nurse shouldn't document for another nurse, except for an authorized entry in an emergency. Omission is a documentation error in which information is missing from the medical record. In this scenario, the day-shift nurse omitted documenting the administration of pain medication. A late entry refers to an entry made later than it should have been. The nurse should identify a necessary late entry as a "late entry" and document the reference date and time. An improper correction is an entry corrected in an improper manner, such as by erasing, using correction fluid, or obliterating the error with a marking pen. The nurse should always follow the facility's documentation guidelines.

The client asks about a new medication, it's side effects, cost and if the drug is compatible with the other medication the client takes. The nurse answers all questions the client asks without withholding information. The nurse is guided by which ethical principle? o beneficence o veracity o fidelity o justice

o veracity · Veracity is the duty to be honest or truthful. The nurse is exercising veracity when fully answering any questions the client is answering without withholding information. Justice, beneficence and fidelity are not the ethical principle described in this question.

A nurse saw a coworker steal drugs from a locked cabinet. The supervisor notices the missing drugs and has a good idea who is responsible for the theft. The supervisor asks if the nurse saw anything out of the ordinary. Which professional value reflects a nurse's duty to tell the truth? o autonomy o veracity o beneficence o nonmaleficence

o veracity · Veracity is the nurse's duty to tell the truth in all professional situations. Autonomy refers to a client's right to self-determination. Beneficence is the duty to do good for the clients assigned to the nurse's care. Nonmaleficence is the duty to do no harm to the client.


Ensembles d'études connexes

RNA Splicing, Capping, Poly-A tail

View Set

AAPC Chapter 14 Practical Application

View Set

WEEK 3: Domain 6 Confidentiality Quiz

View Set

Developmental Psychology Test 1 (ch. 1-6)

View Set